Chương 4: BẤT ĐẲNG THỨC, BẤT PHƯƠNG TRÌNH

Nguyễn Hoàng Minh

Cho a, b, c >0. Chứng minh rằng:

\(\dfrac{1}{a^3+b^3+abc}+\dfrac{1}{b^3+c^3+abc}+\dfrac{1}{c^3+a^3+abc}\le\dfrac{1}{abc}\)

Lightning Farron
30 tháng 12 2017 lúc 12:05

Ta có BĐT:

\(a^3+b^3=\left(a+b\right)\left(a^2+b^2-ab\right)\)\(\ge\left(a+b\right)ab\)

\(\Rightarrow a^3+b^3+abc\ge ab\left(a+b+c\right)\)

\(\Rightarrow\dfrac{1}{a^3+b^3+abc}\le\dfrac{1}{ab\left(a+b+c\right)}\)

Tương tự cho 2 BĐT còn lại rồi cộng theo vế:

\(VT\le\dfrac{1}{ab\left(a+b+c\right)}+\dfrac{1}{bc\left(a+b+c\right)}+\dfrac{1}{ac\left(a+b+c\right)}\)

\(=\dfrac{a+b+c}{abc\left(a+b+c\right)}=\dfrac{1}{abc}=VP\)

Khi \(a=b=c\)


Các câu hỏi tương tự
Anh Huy Nguyễn Hoàng
Xem chi tiết
Anxiety
Xem chi tiết
Thảo Vi
Xem chi tiết
loancute
Xem chi tiết
Rồng Xanh
Xem chi tiết
Vũ Như Quỳnh
Xem chi tiết
Nguyễn Trần
Xem chi tiết
Tae Tae
Xem chi tiết
Nguyen Ha
Xem chi tiết